You are on page 1of 14

PrepTest 74

PrepTest 74
Questions 1–5

Setup:
oo six-member band: G K P S T V
oo each member performs exactly one solo

Conditions:
#1: ~G4
#2: P — K
#3: V — K — G
#4: P — S — T or T — S — P

Overview:
A glance through the rules reveals an early point of connection. Rules 2 and 3 share the variable K. We’ll
combine them into the following:
P
K G
V

This covers four of the six variables. The fourth rule brings in the last two: S and T. Since there are two
possible orderings for P, S, and T, we can create two molds and move straight to the questions.
#1: P S T #2: T S P
K G K G
V V

1. (C) As usual, we get an elimination question to start things off. The first rule eliminates choice A. The
second rule eliminates choices D and E. The third rule doesn’t eliminate any of the remaining choices,
but the fourth rule eliminates B. This leaves C as the correct answer.
(A) breaks the first rule
(B) breaks the fourth rule
(C) Correct
(D) breaks the second rule
(E) breaks the second rule

2. (D) This question triggers mold #1. Other than that, nothing obvious jumps out, so let’s head to the
answer choices.
(A) V could also be first, so this doesn’t have to be true.
(B) P could be first, so this need not be true.
(C) Since we don’t have a one-direction path between V and S, this outcome is not guaranteed.
(D) Correct. Note that this is merely a proper interpretation of the fourth rule.
(E) The ordering relationship between S and K is not solidified in this mold.

Logic Games Solutions -1- ©Cambridge LSAT 2015


PrepTest 74 PrepTest 74
3. (E) The question gives us no new information, so we’ll head straight to the choices.
(A) As our molds show, K must be preceded by at least two other variables.
(B) G must be preceded by at least three others, and it can’t be fourth (first rule), so it can be fifth at
the earliest.
(C) At first glance, mold #1 makes it look like this could happen. However, if G came before S, then
G would also be followed by T, which would push G into the fourth position (remember the
first rule?).
(D) Both molds show that G must come later than P.
(E) Correct. This could be true of mold #1, as we don’t have a definitive ordering between K and S.

4. (A) To exclude a variable from the third position, we need to know one of two things: it must be
preceded by at least three others or it must be followed by at least four others.
(A) Correct. As we established in question 3, choice B, the earliest position G can occupy is fifth.
(B) K only needs to have at least one variable preceding it and at least one variable following it, so
this works.
(C) As mold #1 shows, S only needs to have at least one variable in front of it and one behind it.
(D) T could be third in mold #1; it only needs to be preceded by P and S.
(E) V could be third under either of the molds, so this choice is incorrect.

5. (E) Finally, we get a question which definitively places one of the variables! Placing V fourth pushes K
and G into the fifth and sixth slots, respectively. We’re only left with P, S, and T. The fourth rule gave
us two options, and in either case, S must occupy the second slot. Now, let’s look for the choice which
doesn’t have to be true.

P S T V K G
T S P V K G
1 2 3 4 5 6
(A) P is either first or third, so this must be true.
(B) Like P, T must be either first or third, so this must be true as well.
(C) G is last, so all the other variables precede it.
(D) S is second and V is fourth, so this must be true.
(E) Correct. This need not be true, as the first solution shows.

Questions 6–10

Setup:
oo four art historians: F G H J
oo four lectures: L O S W
oo lectures given one at a time
oo each art historian gives a different lecture

Logic Games Solutions -2- ©Cambridge LSAT 2015


PrepTest 74 PrepTest 74
Conditions:
#1: O
L
W
#2: F — O
#3: G
H
J

Overview:
Right off the bat, it’s clear that we can combine the first and second rules into a larger ordering map since
both rules include O. Here’s what we have:
F O
L
W

That’s pretty useful, but there’s an even larger inference that we can make. This 4-variable ordering map,
by nature of the game’s structure, must span at least three of the game’s ordered positions. We can push the
variables towards the front of the diagram, have F and L touch the two ends of the diagram, and push the
variables against the back of the diagram. That will give us three scenarios which we can flesh out.

#1: W O L S #2: L #3: S/W W/S O L


F H G/J J/G F H G/J J/G H F G/J J/G
1 2 3 4 1 2 3 4 1 2 3 4
~O

Mold #1: With F in the first position, and O and L in the second and third positions, respectively, the
third rule forces us to place H second, and split G and J between the third and fourth slots. W must also
precede L, so we must put it first, leaving S to occupy the fourth slot.
Mold #2: With F in the first position, and L in the last position, we can’t infer anything else definite about
the lectures, aside from the fact that O can’t be given first (second rule). We can, however, place H
second and split G and J between the third and fourth positions, due to the third rule.
Mold #3: With F in the second position, and O and L in the third and fourth positions, the third rule
forces us to place H in the first position, and to split G and J between the third and fourth positions.
Finally, with only the S and W lectures left to assign, they must be placed in the first two positions, in
either order.

6. (E) Naturally, the game kicks off with an orientation question. The first rule eliminates choice A, and
the second rule eliminates choice D. The third rule eliminates choice B and choice C. This leaves E as
the correct answer.

Logic Games Solutions -3- ©Cambridge LSAT 2015


PrepTest 74 PrepTest 74
(A) breaks the first rule
(B) breaks the third rule
(C) breaks the third rule
(D) breaks the second rule
(E) Correct.

7. (B) With no new information, we’ll have to check the answer choices against our molds.
(A) As mold #2 shows, F could give the S lecture.
(B) Correct. This is true of all three molds.
(C) As molds #1 and #2 illustrate, this need not be true.
(D) G and J are interchangeable in all our scenarios, so for the same reason as choice C, this is
incorrect.
(E) This need not be true of mold #2, so it is incorrect.

8. (E) W can only be third under mold #2, so let’s place it there and see what happens. O must be
scheduled after F (second rule), so we’ll place it in the second position. The only spot left for S is first,
which means there are two viable solutions for this scenario. Now, on to the choices!

S O W L
F H G/J J/G
1 2 3 4
(A) F is in the first position and W is in the third position, so this must be false.
(B) O is second, and G can be no earlier than third.
(C) G can be no earlier than third, and S is in the first position.
(D) H has to give the O lecture in the second position.
(E) Correct. As is typically the case, the correct answer touches on the only remaining uncertainty:
the assignment of G and J.

9. (A) We’re not given any new information, so we’ll just check the answers against our scenarios.
(A) Correct. In all three molds, F and L are in different positions.
(B) This could be true of molds #1 and #2.
(C) Our diagram for question 8 proves this answer choice incorrect.
(D) This is true of mold #1 and could be true of mold #2.
(E) This could be true of mold #2.

10. (A) This could be true of molds #1 and #2. In the first scenario, the outcome is clear: J would be third
and give the L lecture. Let’s see what happens with mold #2. J has to give the fourth lecture, and the
second rule dictates that we place O in the second lecture slot, leaving W to occupy the first slot.

W O S L
F H G J
1 2 3 4

Logic Games Solutions -4- ©Cambridge LSAT 2015


PrepTest 74 PrepTest 74
(A) Correct. This corresponds to the result when mold #1 is in effect.
(B) O must be given second in both fleshed-out options.
(C) S has to be given either third or fourth.
(D) Again, S has to be given either third or fourth.
(E) W is first in each of the two options.

Questions 11–16

Setup:
oo three rugs
oo six colors available: F O P T W Y
oo exactly five colors used
oo each used color is in only one rug
oo each rug is solid or multicolored

Conditions:

W
#1: W

O
P
#2: O ; ~P ~O (contrapositive)

F
T
#3:

P
T
#4:

P
Y
#5:

Overview:
The setup for this game is pretty involved. At first glance, it may not be clear what the most efficient way
to diagram everything. It may be tempting to assign a solid/multicolored variable to each rug. However,
glancing at the first question reveals that the only thing we need to know about each rug is its color

Logic Games Solutions -5- ©Cambridge LSAT 2015


PrepTest 74 PrepTest 74
composition. There’s no need to use extra variables, since any rug with two or more colors is multicolored.
The setup conditions establish that we’re going to use 5 of the 6 variables, so we know that any acceptable
solution will have exactly one variable out. Let’s spend some time digesting the second rule. Since any
rug with O has P, we can safely say that if none of the rugs had P, none of them would have O (the
contrapositive). However, that would be a problem, because then we would only have four colors among
the rugs, which would violate the setup conditions. Thus, we can infer that P must always be in no matter
what.

Let’s think about the numbers involved with the game. The rules didn’t establish a maximum number of
colors for any one rug. However, we do know that each rug must have at least one color. Since we are going
to use five colors for each solution, the most we can have in one rug is three. Otherwise, one of the other
two rugs wouldn’t have a color. The only other acceptable way the numbers can work out is if we take one
of the colors from the three-color rug and give it to one of the other two, giving a 2, 2, 1 allocation. Here
are our two acceptable allocations of colors to rugs:
Rug Rug Rug
3 1 1
2 2 1

Other than these deductions, there’s not much else to infer. The only other thing we know for sure that if a
problem addresses the second allocation, W has to be out, due to the first rule. I’m going to label the rugs
in the diagrams with the number of colors, but you don’t have to label them at all if you prefer.

11. (A) The game kicks off with a standard orientation question. The first rule eliminates choice E, and
the second rule eliminates choice C. The third rule eliminates choice D, but the fourth rule doesn’t
eliminate any of the remaining choices. Finally, the fifth rule eliminates B, leaving A as the correct
answer.
(A) Correct.
(B) breaks the fifth rule
(C) breaks the second rule
(D) breaks the third rule
(E) breaks the first rule

12. (C) The only inferences we made were about P always being in, the allocations of colors to rugs, and
that W is out for the 2, 2, 1 allocation. Chances are good that the correct answer will address one of
these. Sure enough, choice C is a restatement of our P inference, and it’s correct.
(A) F can’t be with P (third rule), but what’s to prevent it from being with other colors?
(B) T can’t be with F or P (third and fourth rules), but what’s to prevent it from being with other
colors?
(C) Correct. Excluding P would also exclude O (second rule), which would violate the setup
condition about using five of the six colors.
(D) No, T can be out without causing any problems. If anything, it’d make things easier because then
we wouldn’t have to worry about the third and fourth rules.
(E) Y can be out. Then, we wouldn’t have to worry about satisfying the fifth rule.

Logic Games Solutions -6- ©Cambridge LSAT 2015


PrepTest 74 PrepTest 74
13. (E) This question is tricky. If we have one rug that is solid P, what else can we deduce? P is connected
to O in the second rule. We can’t have more than one of any color, as specified by the setup
conditions. Thus, with only one P, we can infer that O must be out. With O out, all the other colors
must be in, including W. Therefore, we’re in a 3, 1, 1 situation (remember the first rule?). We’ll start
our diagram by putting P in one of the single slot rugs and putting W in the three-slot rug. What else
do we know? F and T can’t be together due to the third rule, so we’ll have to split them between the
remaining rugs with open slots. With only one slot remaining in the three-color rug, we have to place
Y there. Here’s our completed diagram:

W
Y
F/T T/F P
3 1 1
(A) This doesn’t have to be true; the F could be assigned to the three-color rug.
(B) F and T are interchangeable here, so this is incorrect for the same reason as choice A.
(C) As our diagram shows, this isn’t possible; Y must be assigned to the three-color rug.
(D) T could be assigned to the three-color rug, so this doesn’t have to be true.
(E) Correct. W and Y have to be together, as our diagram illustrates.

14. (D) This question is designed to eat up a lot of time. It limits us to the 3, 1, 1 allocation, but we don’t
know which variable is out. We do have two solutions in which there are two solid rugs, so it’d be
prudent to check them against the choices. Our diagram for question 13 proves that A is acceptable,
so we’ll elminate it. Aside from running some scenarios with W in and W out, which would be time
consuming, our only option is to jump in and test the remaining choices.
(A) See our diagram for question 13.
(B) This is easy to prove incorrect. After assigning F and Y, we can assign W, O, and P to the three-
color rug and be done with it.

W
O
P F Y
3 1 1
(C) With P occupying one of the single slots, we know that O has to be out. Otherwise, we’d have
a problem with the second rule. All three remaining variables (F, W, and Y) would have to be
assigned to the three-color rug. This doesn’t break any rules, so the choice is incorrect.

F
W
Y P T
3 1 1

Logic Games Solutions -7- ©Cambridge LSAT 2015


PrepTest 74 PrepTest 74
(D) Correct. As in the previous question, assigning P to a solid rug means that O has to be out. If we
were to assign the remaining variables (F, T, and W) to the three-color rug, we would break the
third rule.

F
T
W P Y
3 1 1
(E) After assigning T and Y to the solid rugs, let’s fill out the three-color rug. W doesn’t conflict with
any of the others, so it’s an easy first choice. We know that O and P can be together (second
rule), so we’ll fill the remaining slots with them. We have a working solution, so this answer
choice is incorrect.

W
O
P T Y
3 1 1

15. (B) Unfortunately, this bit of information doesn’t nail down the allocation of colors to rugs. We’ll have
to dig deeper. What happens when we assign the FP piece to a three-color rug? The third, fourth, and
fifth conditions dictate that T and Y cannot be assigned to that same rug. The remaining slot in the
three-color rug must be filled by either O or W. Whichever one of O and W doesn’t occupy that slot
is out for the solution (O because of the second rule, and W because of the first rule). With only two
variables remaining (T and Y), we can confidently place them in the remaining solid slots.

O/W
F
P T Y
3 1 1
~T
~Y
What happens when we assign the FP piece to a two-color rug? This poses a problem. With a P placed,
and no more remaining slots in the rug, O must be out (contrapositive of the second rule). We’re left
with T, W, and Y, but W also must be out, due to the first rule. Thus, this doesn’t work, and we’ll have
to stick with the 3, 1, 1 allocation and our initial diagram.

F
P
2 2 1

Logic Games Solutions -8- ©Cambridge LSAT 2015


PrepTest 74 PrepTest 74
(A) This must be false. As we’ve established, we’re working with the 3, 1, 1 allocation.
(B) Correct. W can be out, and we’d still have an acceptable solution.
(C) As our diagram illustrates, Y must be assigned in this scenario.
(D) T comprises a solid-color rug, so this must be false.
(E) T and Y both comprise solid-color rugs, so this must be false.

16. (A) As in the previous question, this one doesn’t explicitly target one of the allocations. We now
have some previous work from which to draw, so let’s see if we can eliminate any of the choices. Our
diagram for 14-B disproves choices B and C. Our diagram for question 15 dispenses with choices
D and E, leaving A as the correct choice. Let’s examine why A presents a problem. With the 2, 2, 1
allocation in effect, we know that W is out. With all the remaining variables in, there would have to be
an OP piece (second rule). However, that would leave F and T to fill the slots in the remaining two-
color rug, but they don’t get along (third rule).

O
P Y
2 2 1

(A) Correct. As shown, this scenario would force us to break at least one rule.
(B) See the diagram for 14-B.
(C) See the diagram for 14-B.
(D) See the diagram for question 15.
(E) See the diagram for question 15.

Questions 17–23

Setup:
oo at least two photographers assigned to each ceremony
oo two ceremonies: S T
oo six available photographers: F G H K L M
oo not all have to be assigned
oo assigned photographers are limited to one ceremony

Conditions:

F
H
#1:

L
M
#2:

Logic Games Solutions -9- ©Cambridge LSAT 2015


PrepTest 74 PrepTest 74
#3: GS LT; ~LT ~GS (contrapositive)

F
H
#4: KT or T

Overview:
With three conditionals in the rules, there’s a lot to digest here. First, let’s think about the numbers. There
are six photographers, and at least two of them are assigned to each ceremony. This means that as many
as 2 of them can be out in any acceptable scenario. Another possibility is that all 6 are assigned, so long
as the other rules aren’t broken. Being that only one of the rules is not conditional (the first), and even it
doesn’t definitively place anything anywhere, we’re going to jot down a quick diagram and head right to the
questions.

S 2–4 T 2–4 Out 0–2


G L
~K

H
M

17. (E) Naturally, the game kicks off with an orientation question. The first rule eliminates choice B, and
the second rule eliminates choice C. The third rule eliminates choice A, and the fourth rule eliminates
choice D, leaving E as the correct answer.
(A) breaks the third rule
(B) breaks the first rule
(C) breaks the second rule
(D) breaks the fourth rule
(E) Correct

18. (D) We can immediately combine this with the first rule to create the following piece:

F
H
L

The next thing we need to ask ourselves is where this piece can be assigned. Can it be assigned to S?
Yes, so long as K is assigned to T. The fourth rule requires that either K or HM be assigned to T, so
if we have already assigned H to S, K must go to T. The second T slot will have to be filled by either
G or M. Note that G can’t be assigned to S, as this would trigger and break the third rule. Also, it’s

Logic Games Solutions -10- ©Cambridge LSAT 2015


PrepTest 74 PrepTest 74
possible that both G and M could be assigned to T. As a further inference, M can’t be assigned to S
due to the second rule.

F
H G/M
L K
S T
~G
So, that works. Can we assign the FHL piece to T? Let’s try. Once it’s assigned, we need to check
in on the fourth rule, since it affects so many variables. M can’t be assigned to T because this would
break the second rule; thus we must assign K to T to satisfy the fourth rule. With only two variables
remaining (G and M), we must assign both of them to S. This is our other option.

K
F
G H
M L
S T
~M
(A) This isn’t true of the first option, so we’ll eliminate it.
(B) This isn’t true of the first option, so it’s gone.
(C) This could be true (see the first option), but it doesn’t have to be.
(D) Correct. This is true of both acceptable diagrams.
(E) This isn’t true of the first diagram, so it’s incorrect.

19. (B) As is often the case with questions like this, we must also be aware of the variables which aren’t
listed by the answer choices; they must either be out or assigned to T.
(A) The two unlisted variables (L and M) would have to be assigned to T (recall that each ceremony
needs at least two photographers), but this would break the second rule.
(B) Correct. Let’s try to make this work. The third rule is triggered, so we have to assign L to T.
As a consequence of the fourth rule, K must be assigned to T, since HM won’t be. The only
remaining variable is M, and it can either be assigned to S or left out entirely (due to the second
rule).

Logic Games Solutions -11- ©Cambridge LSAT 2015


PrepTest 74 PrepTest 74

F
G K
H L
S T
~M
(C) This would trigger the third rule, so we’d have to assign L to T. The problem is, the fourth rule
would force us to assign M to T as well (since K is unavailable), which would then break the
second rule.
(D) This breaks the first rule, since F is not grouped with H.
(E) This breaks the fourth rule; at least one of K and M must be assigned to T.

20. (B) Let’s see which answers we can eliminate based on our previous work thus far. We know that
neither one of G and M is required; we established this in diagramming question 18. That allows us to
get rid of choice E. F and H are common to all the remaining choices (and the first rule dictates that
they’re both assigned), so we can ignore them. The next variable we need to test is K. Does K have to
be in? If we leave K out, the fourth rule requires that we assign both H and M to T. The first rule then
requires us to assign F to T. The problem now is that we’re only left with two variables (G and L), and
assigning them both to S would trigger and break the third rule. Therefore, K must always be in.

F
G H
L M
S T
After eliminating choice D (which doesn’t include K), we’ll move on to L. Does L always need to be
assigned? If L is out, we need to avoid triggering the third rule, so we’ll assign G to T. To satisfy the
fourth rule, we’ll also assign K to T, since doing so only takes up one slot. We still need to place F and
H, so we’ll assign them both to S. Now that we have two photographers assigned to each ceremony,
we’re good to go, and we’ve established that L doesn’t have to be in. Note that M could still be assigned
to either ceremony under this scenario, but that doesn’t change anything.

F G
H K
S T
~M
(A) This choice is missing K.
(B) Correct.

Logic Games Solutions -12- ©Cambridge LSAT 2015


PrepTest 74 PrepTest 74
(C) This choice has L, which can be out.
(D) This choice is missing K, and it has G, which can be out.
(E) This choice is missing K, and it has G and M, both of which can be out.

21. (A) We can make quick work of this question by utilizing our diagram for question 20. We have
a working solution with just four photographers assigned, so any which are not assigned to S, are
incorrect. The only one among the choices assigned to S in our diagram is F, and thus, choice A is
correct.
(A) Correct.
(B) G is assigned to T in the diagram.
(C) K is assigned to T in the diagram.
(D) L is out in our solution for question 20.
(E) M could be in or it could be out; either way, it need not be assigned to S.

22. (B) Here’s another question which can take up an inordinate amount of time. Remember that any of
the variables not shown in the choices must either be out or assigned to S. In the case of the first two
choices, the two non-showing variables must be assigned to S, since S needs at least two photographers.
(A) Assigning the remaining variables (K and L) to S doesn’t present any problems, so this is
acceptable. F and H are together (first rule), L and M are assigned to different ceremonies
(second rule), G is not assigned to S (doesn’t trigger the third rule), and H and M are both
assigned to T (fourth rule).

F
G
K H
L M
S T
(B) Correct. If we assign the remaining two variables (G and L) to S, we’ll trigger and break the third
rule, so this must be false.

F
H
G K
L M
S T
(C) With G, K, and L assigned to T, we’ll have to assign F and H to S. M can be assigned to S or not;
either way, this works.

Logic Games Solutions -13- ©Cambridge LSAT 2015


PrepTest 74 PrepTest 74

G
F K
H L
S T
~M
(D) With G, K, and M assigned to T, we’ll have to assign F and H to S. L can be assigned to S or not;
in either case, this is an acceptable outcome.

G
F K
H M
S T
~L
(E) This choice is easy to disprove. As in the previous two choices, we can just assign F and H to S
and be done with it. Next!

F K
H M
S T

23. (C) Finally, we reach the end of the game, and it’s a dreaded equivalent rule question! We need to find
a new rule which ensures that either K or the HM pair is assigned to T. Let’s get to work.
(A) This is incorrect for a lot of reasons. First of all, K doesn’t have to be assigned at all! Accordingly,
we need a condition that covers scenarios in which K is out. Further, this one doesn’t force both
H and M to be assigned to T when K is elsewhere, so it doesn’t work.
(B) This condition also fails in multiple respects. First, it only applies when K is assigned to S, when
it should also apply when K is out. Second, it doesn’t even mention assigning the HM piece to T
when K is not assigned to that group.
(C) Correct. This is merely a restatement of the original. The “unless” wording can be tricky to
interpret, so we’ll address that briefly. You can turn “unless” into “if not” and and not change the
meaning of the original. Thus, this rule means, “If K is not assigned to T, then both K and M
must be.”
(D) Rephrasing the “unless” gives “If K isn’t assigned to T, then H and L cannot be assigned
together.” This condition gets the first half right, but it doesn’t force both H and M to be
assigned to T when K isn’t.
(E) This one is really close. Working with “unless” and “or” in one statement can get pretty messy, so
I’m just going to explain what this choice means. It means that we would need either one of H
and M or K. However, the original condition requires either both H and M or K.

Logic Games Solutions -14- ©Cambridge LSAT 2015

You might also like